am i applying the property correctly??












0












$begingroup$


The time shifting property states that:



$$ mathcal{L}(u(t-t_0)f(t-t_0))=e^{-st_0}F(s) $$



Now I am given a question which is:



$$ mathcal{L}(e^{-4t}u(t-3)) $$



Can I do like I have done here? That the time shift is of 3 seconds, so using the property I get final answer as:



$$ frac{e^{-3s}}{s+4} $$



Edit: if we are asked to find unilateral Laplace Transform then and only then do I get the above answer?










share|cite|improve this question











$endgroup$

















    0












    $begingroup$


    The time shifting property states that:



    $$ mathcal{L}(u(t-t_0)f(t-t_0))=e^{-st_0}F(s) $$



    Now I am given a question which is:



    $$ mathcal{L}(e^{-4t}u(t-3)) $$



    Can I do like I have done here? That the time shift is of 3 seconds, so using the property I get final answer as:



    $$ frac{e^{-3s}}{s+4} $$



    Edit: if we are asked to find unilateral Laplace Transform then and only then do I get the above answer?










    share|cite|improve this question











    $endgroup$















      0












      0








      0





      $begingroup$


      The time shifting property states that:



      $$ mathcal{L}(u(t-t_0)f(t-t_0))=e^{-st_0}F(s) $$



      Now I am given a question which is:



      $$ mathcal{L}(e^{-4t}u(t-3)) $$



      Can I do like I have done here? That the time shift is of 3 seconds, so using the property I get final answer as:



      $$ frac{e^{-3s}}{s+4} $$



      Edit: if we are asked to find unilateral Laplace Transform then and only then do I get the above answer?










      share|cite|improve this question











      $endgroup$




      The time shifting property states that:



      $$ mathcal{L}(u(t-t_0)f(t-t_0))=e^{-st_0}F(s) $$



      Now I am given a question which is:



      $$ mathcal{L}(e^{-4t}u(t-3)) $$



      Can I do like I have done here? That the time shift is of 3 seconds, so using the property I get final answer as:



      $$ frac{e^{-3s}}{s+4} $$



      Edit: if we are asked to find unilateral Laplace Transform then and only then do I get the above answer?







      laplace-transform






      share|cite|improve this question















      share|cite|improve this question













      share|cite|improve this question




      share|cite|improve this question








      edited Dec 30 '18 at 22:10









      Paul Enta

      5,18111334




      5,18111334










      asked Dec 20 '18 at 15:52









      Ahmad QayyumAhmad Qayyum

      677




      677






















          1 Answer
          1






          active

          oldest

          votes


















          1












          $begingroup$

          It's wrong!
          Go step by step.



          Step $1$: Find frequency shift due to $e^{-4t}$ using:



          If $mathcal{L}(f(t))=F(s)$, then
          $mathcal{L}(e^{-4t}f(t))=F(s+4)$. (which is the answer)



          Step $2$: Find $mathcal{L}(f(t))$ by the property of time shift as follows:
          $$mathcal{L}(u(t-3).1)=frac{e^{-3s}}{s}$$
          where $f(t)=u(t-3).1$



          So that finally $$F(s+4)=frac{e^{-3(s+4)}}{s+4}$$






          share|cite|improve this answer











          $endgroup$













          • $begingroup$
            can you tell me why can't apply the property over here? what are the cases where we can apply this property? does those cases involve the unshifted version of unit step functions?
            $endgroup$
            – Ahmad Qayyum
            Dec 25 '18 at 18:19











          Your Answer





          StackExchange.ifUsing("editor", function () {
          return StackExchange.using("mathjaxEditing", function () {
          StackExchange.MarkdownEditor.creationCallbacks.add(function (editor, postfix) {
          StackExchange.mathjaxEditing.prepareWmdForMathJax(editor, postfix, [["$", "$"], ["\\(","\\)"]]);
          });
          });
          }, "mathjax-editing");

          StackExchange.ready(function() {
          var channelOptions = {
          tags: "".split(" "),
          id: "69"
          };
          initTagRenderer("".split(" "), "".split(" "), channelOptions);

          StackExchange.using("externalEditor", function() {
          // Have to fire editor after snippets, if snippets enabled
          if (StackExchange.settings.snippets.snippetsEnabled) {
          StackExchange.using("snippets", function() {
          createEditor();
          });
          }
          else {
          createEditor();
          }
          });

          function createEditor() {
          StackExchange.prepareEditor({
          heartbeatType: 'answer',
          autoActivateHeartbeat: false,
          convertImagesToLinks: true,
          noModals: true,
          showLowRepImageUploadWarning: true,
          reputationToPostImages: 10,
          bindNavPrevention: true,
          postfix: "",
          imageUploader: {
          brandingHtml: "Powered by u003ca class="icon-imgur-white" href="https://imgur.com/"u003eu003c/au003e",
          contentPolicyHtml: "User contributions licensed under u003ca href="https://creativecommons.org/licenses/by-sa/3.0/"u003ecc by-sa 3.0 with attribution requiredu003c/au003e u003ca href="https://stackoverflow.com/legal/content-policy"u003e(content policy)u003c/au003e",
          allowUrls: true
          },
          noCode: true, onDemand: true,
          discardSelector: ".discard-answer"
          ,immediatelyShowMarkdownHelp:true
          });


          }
          });














          draft saved

          draft discarded


















          StackExchange.ready(
          function () {
          StackExchange.openid.initPostLogin('.new-post-login', 'https%3a%2f%2fmath.stackexchange.com%2fquestions%2f3047676%2fam-i-applying-the-property-correctly%23new-answer', 'question_page');
          }
          );

          Post as a guest















          Required, but never shown

























          1 Answer
          1






          active

          oldest

          votes








          1 Answer
          1






          active

          oldest

          votes









          active

          oldest

          votes






          active

          oldest

          votes









          1












          $begingroup$

          It's wrong!
          Go step by step.



          Step $1$: Find frequency shift due to $e^{-4t}$ using:



          If $mathcal{L}(f(t))=F(s)$, then
          $mathcal{L}(e^{-4t}f(t))=F(s+4)$. (which is the answer)



          Step $2$: Find $mathcal{L}(f(t))$ by the property of time shift as follows:
          $$mathcal{L}(u(t-3).1)=frac{e^{-3s}}{s}$$
          where $f(t)=u(t-3).1$



          So that finally $$F(s+4)=frac{e^{-3(s+4)}}{s+4}$$






          share|cite|improve this answer











          $endgroup$













          • $begingroup$
            can you tell me why can't apply the property over here? what are the cases where we can apply this property? does those cases involve the unshifted version of unit step functions?
            $endgroup$
            – Ahmad Qayyum
            Dec 25 '18 at 18:19
















          1












          $begingroup$

          It's wrong!
          Go step by step.



          Step $1$: Find frequency shift due to $e^{-4t}$ using:



          If $mathcal{L}(f(t))=F(s)$, then
          $mathcal{L}(e^{-4t}f(t))=F(s+4)$. (which is the answer)



          Step $2$: Find $mathcal{L}(f(t))$ by the property of time shift as follows:
          $$mathcal{L}(u(t-3).1)=frac{e^{-3s}}{s}$$
          where $f(t)=u(t-3).1$



          So that finally $$F(s+4)=frac{e^{-3(s+4)}}{s+4}$$






          share|cite|improve this answer











          $endgroup$













          • $begingroup$
            can you tell me why can't apply the property over here? what are the cases where we can apply this property? does those cases involve the unshifted version of unit step functions?
            $endgroup$
            – Ahmad Qayyum
            Dec 25 '18 at 18:19














          1












          1








          1





          $begingroup$

          It's wrong!
          Go step by step.



          Step $1$: Find frequency shift due to $e^{-4t}$ using:



          If $mathcal{L}(f(t))=F(s)$, then
          $mathcal{L}(e^{-4t}f(t))=F(s+4)$. (which is the answer)



          Step $2$: Find $mathcal{L}(f(t))$ by the property of time shift as follows:
          $$mathcal{L}(u(t-3).1)=frac{e^{-3s}}{s}$$
          where $f(t)=u(t-3).1$



          So that finally $$F(s+4)=frac{e^{-3(s+4)}}{s+4}$$






          share|cite|improve this answer











          $endgroup$



          It's wrong!
          Go step by step.



          Step $1$: Find frequency shift due to $e^{-4t}$ using:



          If $mathcal{L}(f(t))=F(s)$, then
          $mathcal{L}(e^{-4t}f(t))=F(s+4)$. (which is the answer)



          Step $2$: Find $mathcal{L}(f(t))$ by the property of time shift as follows:
          $$mathcal{L}(u(t-3).1)=frac{e^{-3s}}{s}$$
          where $f(t)=u(t-3).1$



          So that finally $$F(s+4)=frac{e^{-3(s+4)}}{s+4}$$







          share|cite|improve this answer














          share|cite|improve this answer



          share|cite|improve this answer








          edited Dec 20 '18 at 18:19

























          answered Dec 20 '18 at 17:54









          Sameer BahetiSameer Baheti

          5718




          5718












          • $begingroup$
            can you tell me why can't apply the property over here? what are the cases where we can apply this property? does those cases involve the unshifted version of unit step functions?
            $endgroup$
            – Ahmad Qayyum
            Dec 25 '18 at 18:19


















          • $begingroup$
            can you tell me why can't apply the property over here? what are the cases where we can apply this property? does those cases involve the unshifted version of unit step functions?
            $endgroup$
            – Ahmad Qayyum
            Dec 25 '18 at 18:19
















          $begingroup$
          can you tell me why can't apply the property over here? what are the cases where we can apply this property? does those cases involve the unshifted version of unit step functions?
          $endgroup$
          – Ahmad Qayyum
          Dec 25 '18 at 18:19




          $begingroup$
          can you tell me why can't apply the property over here? what are the cases where we can apply this property? does those cases involve the unshifted version of unit step functions?
          $endgroup$
          – Ahmad Qayyum
          Dec 25 '18 at 18:19


















          draft saved

          draft discarded




















































          Thanks for contributing an answer to Mathematics Stack Exchange!


          • Please be sure to answer the question. Provide details and share your research!

          But avoid



          • Asking for help, clarification, or responding to other answers.

          • Making statements based on opinion; back them up with references or personal experience.


          Use MathJax to format equations. MathJax reference.


          To learn more, see our tips on writing great answers.




          draft saved


          draft discarded














          StackExchange.ready(
          function () {
          StackExchange.openid.initPostLogin('.new-post-login', 'https%3a%2f%2fmath.stackexchange.com%2fquestions%2f3047676%2fam-i-applying-the-property-correctly%23new-answer', 'question_page');
          }
          );

          Post as a guest















          Required, but never shown





















































          Required, but never shown














          Required, but never shown












          Required, but never shown







          Required, but never shown

































          Required, but never shown














          Required, but never shown












          Required, but never shown







          Required, but never shown







          Popular posts from this blog

          Aardman Animations

          Are they similar matrix

          Ficheiro:Flag of Oman.svg